3
$\begingroup$

I would like an example where the maximum principle fails in a bounded smooth domain $\Omega$ where one has a solution which is not $C^2(\Omega)$ to $Lu=0$ where $L$ is elliptic and linear. This obviously must rely on the coefficients being discontiuous for the elliptic operator since otherwise one can do interior regularity estimates. All of the examples I have tried to come up with turn out to not actually be weak solutions so I'm stuck on this. Perhaps maximum principles extend to the non-smooth setting? Any insight on this would be appreciated.

$\endgroup$

2 Answers 2

10
$\begingroup$

Yes indeed, the maximum principle extends to the non-smooth setting. I am not sure that there is a complete theory, because so many situations can occur. But at least let me mention the following situations.

  1. In Hopf's maximum principle, one only needs that $u$ admits second order derivatives (this is less than ${\mathcal C}^2$) and the coefficients of $L$ be bounded.
  2. In the variational case, that if $L$ is self-adjoint, the $H^1$ solution is unique and minimizes a functional $\int (Lu,u)dx$. Using the fact that if $v$ is in the Sobolev space $H^1$, then the absolute value $|v|$ is in this space too, one again proves a maximum principle.
  3. The previous item covers the one-dimensional case, because then second order operator can always be multiplied by an appropriate weight in such a way that they become self-adjoint. This is the reason why the spectrum of a one-D second-order operator is real (Liouville's theory).
  4. If $L$ is elliptic with smooth coefficients, and if $u$ is a distribution such that $Lu\in{\mathcal C}^\infty$, then $u\in{\mathcal C}^\infty$. Therefore the maximum principle is valid.
  5. The modern theory of second-order elliptic (not necessarily linear) equations is based upon the maximum principle. One declares that $u\in{\mathcal C}^0$ is a solution if, given an arbitrary point, there exists a super-solution (resp. a sub-solution) $\phi_\pm$ such that $\phi_\pm(x)=u(x)$, and $\phi_+\le u\le \phi_-$ otherwise.
$\endgroup$
1
  • $\begingroup$ Thanks for all of the nice examples and facts Denis! I'll use them to think a bit more about this. $\endgroup$
    – Dorian
    Sep 19, 2010 at 19:52
3
$\begingroup$

Maximum principle, in general, can be applied in the viscosity solutions setting (in this case, the viscosity solution is only continuous). You can have a look at the excellent paper by Crandall-Ishii-Lions as following: http://arxiv.org/abs/math/9207212

$\endgroup$

Your Answer

By clicking “Post Your Answer”, you agree to our terms of service and acknowledge you have read our privacy policy.

Not the answer you're looking for? Browse other questions tagged or ask your own question.